LSAT and Law School Admissions Forum

Get expert LSAT preparation and law school admissions advice from PowerScore Test Preparation.

 Administrator
PowerScore Staff
  • PowerScore Staff
  • Posts: 8937
  • Joined: Feb 02, 2011
|
#37376
Complete Question Explanation

Flaw in the Reasoning. The correct answer choice is (A)

The author suspects that NoSmoke does not reduce smokers’ cravings, contrary to what the
manufacturers of NoSmoke may claim. As evidence, she notes that NoSmoke contains only two
ingredients, one of which has been found not to reduce such cravings. If similar results were found
for the second ingredient, the author concludes, we can be certain that NoSmoke does not reduce
smokers’ cravings.

This argument is flawed because the author’s conclusion is based on a questionable premise—
namely, that the ingredients in NoSmoke would work independently to produce the desired effect.
However, what if they only work in conjunction with one another? The two studies would only be
testing the effect of each ingredient independently. Even if neither of them were able to reduce the
smokers’ cravings individually, it is still possible that the two would work together to produce the
desired effect. This is an example of an Error of Composition.

Answer Choice (A): This is the correct answer choice. This answer choice describes an Error of
Composition. Even if the studies establish that each ingredient fails to produce the desired effect,
this does not preclude the possibility that the two ingredients, when taken together, do produce that
effect.

Answer Choice (B): While the author does draw a causal conclusion, namely, that NoSmoke causes
no decrease in cravings for cigarettes, her conclusion is not based on a mere correlation. The studies
described would suggest that the effect was never produced in the presence of either ingredient.
This is not a correlation. Since one portion of this answer choice is an inaccurate description of the
argument, it is incorrect.

Answer Choice (C): There is no reason to suspect that the studies are based on an unrepresentative
sample. NoSmoke contains only two ingredients. If the author only considered the effects of one
ingredient, then this answer choice would be more attractive. However, the author reaches her
conclusion only after considering the hypothetical results of a second study, in which the second
ingredient produces similar results.

Answer Choice (D): This is a Shell Game answer. The author never suggested that NoSmoke is
unable to help people quit smoking: the conclusion has a much narrower focus on whether or not
NoSmoke reduces cravings for cigarettes. Even if NoSmoke were found to help people quit smoking
in some other way, the author’s conclusion would still be valid.

Answer Choice (E): Although the manufactures of NoSmoke may indeed be biased in favor of their
product, the author never discussed their motivations for claiming that their product is effective. This
answer choice describes a Source argument, which is not present in the stimulus.
 Nina
  • Posts: 81
  • Joined: Sep 11, 2012
|
#5562
For the correct answer A, does it mean, the argument illicitly presumes that if both ingredients do not contain quality of reducing smokers' cravings for cigarettes, then the whole product also lacks quality of reducing smokers' cravings?

I chose B because i thought it provides an alternative way of explanation: if ingredients are not the cause of reducing smokers' cravings, there's mere a correlation between ingredients and the effect of reducing cravings, which also causes the argument to be flawed.

Thanks a lot!
User avatar
 Dave Killoran
PowerScore Staff
  • PowerScore Staff
  • Posts: 5925
  • Joined: Mar 25, 2011
|
#5601
Hi Nina,

You are correct in your interpretation of (A). It's one of those classic whole/part arguments where just because the pieces don't have a certain quality, they falsely presume the whole lacks that same quality.

I see where you are coming from with (B), but the problem is that there isn't an actual confusion of correlation with cause. They use causal language, but they've already stated in a premise that the one ingredient doesn't cause a decrease in cravings (that's not a correlation/causation error; they are allowed to state causal assertions in the premises). With a solid causal premise asserted, we'd need more clear correlation asserted and then a clear causal error to justify this answer.

Please let me know if that helps. Thanks!
 Nina
  • Posts: 81
  • Joined: Sep 11, 2012
|
#5609
Hey Dave,

Thank you for your response! It is very helpful :)

Get the most out of your LSAT Prep Plus subscription.

Analyze and track your performance with our Testing and Analytics Package.